Quantcast
  • Register
PhysicsOverflow is a next-generation academic platform for physicists and astronomers, including a community peer review system and a postgraduate-level discussion forum analogous to MathOverflow.

Welcome to PhysicsOverflow! PhysicsOverflow is an open platform for community peer review and graduate-level Physics discussion.

Please help promote PhysicsOverflow ads elsewhere if you like it.

News

PO is now at the Physics Department of Bielefeld University!

New printer friendly PO pages!

Migration to Bielefeld University was successful!

Please vote for this year's PhysicsOverflow ads!

Please do help out in categorising submissions. Submit a paper to PhysicsOverflow!

... see more

Tools for paper authors

Submit paper
Claim Paper Authorship

Tools for SE users

Search User
Reclaim SE Account
Request Account Merger
Nativise imported posts
Claim post (deleted users)
Import SE post

Users whose questions have been imported from Physics Stack Exchange, Theoretical Physics Stack Exchange, or any other Stack Exchange site are kindly requested to reclaim their account and not to register as a new user.

Public \(\beta\) tools

Report a bug with a feature
Request a new functionality
404 page design
Send feedback

Attributions

(propose a free ad)

Site Statistics

205 submissions , 163 unreviewed
5,047 questions , 2,200 unanswered
5,345 answers , 22,709 comments
1,470 users with positive rep
816 active unimported users
More ...

  Soft Bremsstrahlung: why $\hat{k}\cdot\mathbf{v}= \mathbf{v}'\cdot\mathbf{v}$?

+ 4 like - 0 dislike
1132 views

On page 181 in Peskin & Schroeder they say that we consider the integral (intensity) $$\tag{1}\mathcal{I}(\mathbf{v},\mathbf{v}') = \int\frac{\mathrm{d}\Omega_\hat{k}}{4\pi}\,\frac{2(1-\mathbf{v}\cdot\mathbf{v}')}{(1-\hat{k}\cdot\mathbf{v})(1-\hat{k}\cdot\mathbf{v}')}-\frac{m^2/E^2}{(1-\hat{k}\cdot\mathbf{v}')^2}-\frac{m^2/E^2}{(1-\hat{k}\cdot\mathbf{v})^2}$$ in the extreme relativistic limit (ERL). Then they say that in this limit most of the radiated energy comes from the two peaks in the first term of $(1)$. Is this because in the ERL one can take the mass $m$ to be zero: $m=0 ~(\text{ERL})$ so only the first term in $(1)$ remains?

The next question is what I really want an explanation for: They claim that in (ERL) we break up the integral into a piece for each peak, let $\theta=0$ along the peak in each case. Integrate over a small region around $\theta=0$, as follows: $$\tag{2}\mathcal{I}(\mathbf{v},\mathbf{v}') \approx \int_{\hat{k}\cdot\mathbf{v}= \mathbf{v}'\cdot\mathbf{v}}^{\cos\theta=1}\mathrm{d}\cos\theta\,\frac{(1-\mathbf{v}\cdot\mathbf{v}')}{(1-v\cos\theta)(1-\mathbf{v}\cdot\mathbf{v}')} \\[1cm] +\int_{\hat{k}\cdot\mathbf{v}'= \mathbf{v}'\cdot\mathbf{v}}^{\cos\theta=1}\mathrm{d}\cos\theta\,\frac{(1-\mathbf{v}\cdot\mathbf{v}')}{(1-v'\cos\theta)(1-\mathbf{v}\cdot\mathbf{v}')}. $$

Then they claim that the lower limit are really not that important, but in any case: my question is where the lower limits comes from and how about the replacement in the denominator of the integrand, in other words: How does one go from $(1)$ to $(2)$?

I should add that $\mathbf{v}, \mathbf{v}'$ are the particle velocity before and after interaction. I think one must have access to the book to understand the question unfortunately, other than that, I just want to understand where the lower limits of the integral comes from.

NOTE: PS are working in a frame where $p^0=p^{'0}=E$ which (according to them) implies $$k^\mu=(k,\mathbf{k}),~~p^\mu=E(1,\mathbf{v}),~~p^{'\mu}=E(1,\mathbf{v'}) $$ where (I guess) $k=||\mathbf{k}||. $ Then for instance $(k_\mu p^\mu)^2$ becomes $(Ek)^2\left(1-\frac{\mathbf{k}}{k}\cdot\mathbf{v}\right)^2$ which is (I assume) one of the denominators (up so some factors) in $(1)$. So I guess the correct notation in $(1)$ should be $$\tag{3}\mathcal{I}(\mathbf{v},\mathbf{v}') =\int \dots-\frac{m^2/E^2}{\left(1-\hat{\mathbf{k}}\cdot\mathbf{v}'\right)^2}-\frac{m^2/E^2}{\left(1-\hat{\mathbf{k}}\cdot\mathbf{v}\right)^2}. $$

Overall, bad notation is used IMO on the pages near 181 in PS.

This post imported from StackExchange Physics at 2014-03-30 03:09 (UCT), posted by SE-user Love Learning
asked Feb 23, 2014 in Theoretical Physics by Love Learning (165 points) [ no revision ]

1 Answer

+ 1 like - 0 dislike

I am not sure if my answer is correct, from what I understood:

(i) At the relativistic limit, $m<<E$, so the second and third terms in (6.15) will be negligible, just as you said.

(ii) P&S is aiming at $\hat{k}$ parallel to $\mathbf{v}$ or $\mathbf{v}'$ and integrating around $\theta=0$, since (6.15) peaks there (also ref my comment below this answer). For the lower limit of integration in the first term in your Eq. (2), $\hat{k}$ is parallel with $\mathbf{v}'$. Thus $$\hat{k} \cdot \mathbf{v} = v \cos \theta \approx \cos \theta $$, since at the relativistic limit, $v \approx 1$ (ref bottom of p180). And $$\mathbf{v}' \cdot \mathbf{v}= v' v \cos \theta \approx \cos \theta$$. So the lower limit is valid for any $\theta$.

For the lower limit in the second term in your Eq. (2), $\hat{k}$ is parallel to $\mathbf{v}$. Similar with the first term, the lower limit is valid for any $\theta$ as well.

(iii) Once $$\hat{k} \cdot \mathbf{v} = \mathbf{v}' \cdot \mathbf{v} $$, we replace $ 1- \hat{k} \cdot \mathbf{v} $ as $1- \mathbf{v}' \cdot \mathbf{v} $, we will get $$ \frac{ 1-\mathbf{v}' \cdot \mathbf{v} }{ ( 1- v' \cos \theta)( 1- \mathbf{v}' \cdot \mathbf{v} ) } $$. At the relativistic limit, $v \approx v' \approx 1$. I think we can use $v$ and $v'$ interchangeably. Since P&S is aiming at a small region around $\theta =0$, the integrand will not change. Namely our replacement is valid in this small region.

This post imported from StackExchange Physics at 2014-03-30 03:09 (UCT), posted by SE-user user26143
answered Feb 23, 2014 by user26143 (405 points) [ no revision ]
Thanks, I'll go through your answer tomorrow.

This post imported from StackExchange Physics at 2014-03-30 03:09 (UCT), posted by SE-user Love Learning
OK but can we just choose as we wish to have $k$ parallel to $v'$ and then parallel to $v$? What is the physical interpretation? Could you please elaborate a little on that?

This post imported from StackExchange Physics at 2014-03-30 03:09 (UCT), posted by SE-user Love Learning
It's quite confusing here. From what I understood, as P&S said under (6.16), "The integrand of $\mathcal{I}(\mathbf{v},\mathbf{v}')$ peaks when $\hat{k}$ is parallel to either $\mathbf{v}$ or $\mathbf{v'}$". The denominator in the first term of (6.15) is $(1-\hat{k} \cdot \mathbf{v} ) (1-\hat{k} \cdot \mathbf{v}' ) =(1-v \cos \theta_{kv} ) (1-v' \cos \theta_{kv'} ) \approx (1- \cos \theta_{kv} ) (1- \cos \theta_{kv'} )$ . It has to be either $\theta_{kv}=0$ or $\theta_{kv'}=0$ for the peaks.

This post imported from StackExchange Physics at 2014-03-30 03:09 (UCT), posted by SE-user user26143
I think the book should explain better, but thanks for your answer.

This post imported from StackExchange Physics at 2014-03-30 03:09 (UCT), posted by SE-user Love Learning

Your answer

Please use answers only to (at least partly) answer questions. To comment, discuss, or ask for clarification, leave a comment instead.
To mask links under text, please type your text, highlight it, and click the "link" button. You can then enter your link URL.
Please consult the FAQ for as to how to format your post.
This is the answer box; if you want to write a comment instead, please use the 'add comment' button.
Live preview (may slow down editor)   Preview
Your name to display (optional):
Privacy: Your email address will only be used for sending these notifications.
Anti-spam verification:
If you are a human please identify the position of the character covered by the symbol $\varnothing$ in the following word:
$\varnothing\hbar$ysicsOverflow
Then drag the red bullet below over the corresponding character of our banner. When you drop it there, the bullet changes to green (on slow internet connections after a few seconds).
Please complete the anti-spam verification




user contributions licensed under cc by-sa 3.0 with attribution required

Your rights
...